- PowerScore Staff
- Posts: 5972
- Joined: Mar 25, 2011
- Sun Jan 01, 2012 3:16 pm
#82686
Complete Question Explanation
(The complete setup for this game can be found here: lsat/viewtopic.php?t=867)
The correct answer choice is (A).
Answer choices (A) is possible when the rounds are odd-even: when 3 plays 2, J first defeats S and holds position, and then when 2 plays 1, J defeats R and rises to position 1. Thus, answer choice (A) is possible and correct.
Answer choices (B) and (C) are the same type of answer: a player at one of the “ends” trying to win or lose two matches. This is not possible due to the way the matches leave a player out each time. As the position 5 player, if L plays in the first round, it must be an odd round. If L loses the first match, then L would not play in the second round,which would be even. Similarly, as the position 1 player, if R plays in the first round, it must be an even round. If R wins the first match, then R would not play in the second round,which would be odd.
Answer choice (D) is incorrect because J and L are too far apart for L’s only match to be against J.
Answer choice (E) is incorrect since there cannot be consecutive identical matches as part of the pattern of the game.
(The complete setup for this game can be found here: lsat/viewtopic.php?t=867)
The correct answer choice is (A).
Answer choices (A) is possible when the rounds are odd-even: when 3 plays 2, J first defeats S and holds position, and then when 2 plays 1, J defeats R and rises to position 1. Thus, answer choice (A) is possible and correct.
Answer choices (B) and (C) are the same type of answer: a player at one of the “ends” trying to win or lose two matches. This is not possible due to the way the matches leave a player out each time. As the position 5 player, if L plays in the first round, it must be an odd round. If L loses the first match, then L would not play in the second round,which would be even. Similarly, as the position 1 player, if R plays in the first round, it must be an even round. If R wins the first match, then R would not play in the second round,which would be odd.
Answer choice (D) is incorrect because J and L are too far apart for L’s only match to be against J.
Answer choice (E) is incorrect since there cannot be consecutive identical matches as part of the pattern of the game.
Dave Killoran
PowerScore Test Preparation
Follow me on X/Twitter at http://twitter.com/DaveKilloran
My LSAT Articles: http://blog.powerscore.com/lsat/author/dave-killoran
PowerScore Podcast: http://www.powerscore.com/lsat/podcast/
PowerScore Test Preparation
Follow me on X/Twitter at http://twitter.com/DaveKilloran
My LSAT Articles: http://blog.powerscore.com/lsat/author/dave-killoran
PowerScore Podcast: http://www.powerscore.com/lsat/podcast/